Đến nội dung

Minhnguyenthe333 nội dung

Có 788 mục bởi Minhnguyenthe333 (Tìm giới hạn từ 26-05-2020)



Sắp theo                Sắp xếp  

#642781 $p^{p+1}+(p+1)^p \mid (p^2+p)^{p+1}-1$

Đã gửi bởi Minhnguyenthe333 on 29-06-2016 - 15:47 trong Số học

(Aops) Tìm tất cả các số $p\in \mathbb{P}$ sao cho: $\frac{(p^2+p)^{p+1}-1}{p^{p+1}+(p+1)^p}$ là số nguyên dương




#642761 $Max$ của $T_{n}=a_{1}.a_{2}.......

Đã gửi bởi Minhnguyenthe333 on 29-06-2016 - 12:29 trong Bất đẳng thức - Cực trị

Phân tích số $2014$ thành tổng của $n(n\geq 2)$ số nguyên dương $a_{k}$ với $a_{k}\geq 2$ $(k=\overline{1;n})$. Hãy tìm $GTLN$ của tích sau: $T_{n}=a_{1}.a_{2}.....a_{n}.$

 

Giả sử $c\geqslant b\geqslant a$

Nếu $M$ đạt $GTLN$ khi $c-a\geqslant 2:$

Thay $c\rightarrow c-1$ và thay $a\rightarrow a+1$

$=>(c-1)(a+1)=ca+c-a-1>ca$

Do đó ta nhận được tích khác lớn hơn tích ban đầu

 

Suy ra $M$ đạt $GTLN$ khi $0\leqslant c-a\leqslant 1$.Như vậy $3$ số $a,b,c$ chỉ có thể bằng nhau hoặc nhận 2 giá trị liên tiếp $m,m+1$

Giả sử có $k$ số bằng $m+1$ và $3-k$ số bằng $m$ $(0\leqslant k<3)$

 

$=>P\leqslant m^{3-k}(m+1)^k$

 

Do $a+b+c=100<=>k(m+1)+m(3-k)=100<=>k+3m=100$

Với $m=\left \lfloor \frac{100}{3} \right \rfloor=33$ suy ra $k=1$

 

Vậy $P_{max}=33^2.34=37026$ khi $(a,b,c)=(33,33,34)$ và hoán vị

 

Từ bài này có thể tổng quát thành bài trên của bạn:

$T_n\leqslant m^{n-k}(m+1)^k$ với $m=\left \lfloor \frac{2014}{n}  \right \rfloor$ và $k=2014-mn$




#642735 Giải phương trình nghiệm tự nhiên: $7x^2+9y^2=16z^2$

Đã gửi bởi Minhnguyenthe333 on 29-06-2016 - 09:22 trong Số học

Giải phương trình nghiệm tự nhiên: $7x^2+9y^2=16z^2$ 

 

Tổng quát: Giải phương trình nghiệm tự nhiên $ax^2+by^2=(a+b)z^2$ $(a,b$ là hệ số$)$




#642643 $\sum_{i=1}^n x_i^2>\frac{1}{4...

Đã gửi bởi Minhnguyenthe333 on 28-06-2016 - 18:13 trong Bất đẳng thức và cực trị

Cho dãy giảm $n$ số dương $x_1,x_2,x_3,...,x_n$ thỏa mãn điều kiện $x_1+x_2+....+x_k\geqslant \sqrt{k}$ $(n\geqslant k)$.Chứng minh rằng: $x_1^2+x_2^2+...+x_n^2>\frac{1}{4}(1+\frac{1}{2}+...+\frac{1}{n})$




#642642 $p+q=r(p-q)^n$

Đã gửi bởi Minhnguyenthe333 on 28-06-2016 - 18:06 trong Số học

Bạn hiểu sai ý rồi   :)

$p+q=r(p-q)^{n}$. $p\equiv q(mod 3)\Rightarrow 3|p+q$

Ừa mình hiểu rồi




#642640 $p+q=r(p-q)^n$

Đã gửi bởi Minhnguyenthe333 on 28-06-2016 - 17:52 trong Số học

Dễ thôi

Dễ thấy $ p \geq q $

Giả sử $p,q \geq 3 $

Khi đó do $ 3 \nmid p+q => p \equiv q $ ( mod $3$)

Do đó $p-q \vdots 3 => p+q \vdots 3 $ vô lý

Do đó $p \vdots 3 $ hoặc $q \vdots 3$

TH1: $p= 3 => q=2 => r=5; n \in N $

TH2: $q=3$

TH2.1: $ p \geq 11 => p+3 = r(p-3)^n \geq r(p-3) => 1\leq r \leq \frac{p+3}{p-3} <2 $

Do đó $r=1 => p+3=(p-3)^n$

TH2.2: $ p \leq 7 $

Rồi xét $p$ là xong 




#642468 $y^{2}=x^{3}+16.$

Đã gửi bởi Minhnguyenthe333 on 27-06-2016 - 17:41 trong Số học

Giải phương trình nghiệm nguyên sau: $y^{2}=x^{3}+16.$

Nếu $x$ lẻ và có dạng $4k+1,4k+3$ thì $LHS\equiv 17,19$ $(mod$ $4)$ nên ta loại $TH$ này

 

Do đó $x$ chẵn kéo theo $y$ chẵn$=>PT<=>2x_1^3+4=y_1^2$

 

Suy ra $y_1=2y_2=>PT$ $<=>x_1^3+2=2y_2^2$

 

Kéo theo $x_1=2x_2=>PT$ $<=>4x_2^3=y_2^2-1$

 

Suy ra $y_2=2y_3+1$ nên $PT<=>x_2^3=y_3(y_3+1)$

 

Nhận thấy $y_3(y_3+1)$ là tích 2 số nguyên liên tiếp$=>y_3=0$ và $x_2=0$

 

Hay $(x,y)=(0,\pm 4)$




#642408 Tìm tất cả các số nguyên dương lẻ $n$ sao cho $n\mid 15^n...

Đã gửi bởi Minhnguyenthe333 on 27-06-2016 - 09:57 trong Số học

Mình nghĩ bài toán tổng quát sau đây cũng đúng: Cho $m$ là một số nguyên dương, $m>1$. Tìm tất cả các số nguyên dương lẻ $n$ để $n\mid (2^m-1)^n+1$.

Mình giải như sau:

Ta thấy $n=1$ thỏa mãn

Xét $n>1:$

Gọi $p$ là ước nguyên tố nhỏ nhất của $n$ và gọi $k$ là số nguyên dương nhỏ nhất để $p\mid (2^m-1)^k-1$

Áp dụng định lý Fermat nhỏ: $(2^m-1)^{p-1}-1 \equiv 0$ $(mod$ $p)$

Mặt khác: $(2^m-1)^{2n}-1=[(2^m-1)^n-1][(2^m-1)^n+1]$ nên $p \mid (2^m-1)^{2n}-1$

Do $k$ là số nguyên dương nhỏ nhất nên $k\mid 2n$ và $k\mid p-1$

 

$TH1:k$ lẻ$=>k \mid n$ mà $k\leqslant p-1<p$

$=>k=1$ kéo theo $p \mid 2^m-2$ 

$TH2:k$ chẵn$=>k\mid n$, tương tự như trên $=>k=2$ kéo theo $p\mid 2^m(2^m-2)<=>p\mid 2^m-2$

 

Vậy cả 2 trường hợp đều cho $p\mid 2^m-2$

 

Từ giả thiết ta có: $(2^m-1)^n+1\equiv 0$ $(mod$ $p)$

Mà do $p\mid 2^m-2$ nên $(2^m-1)^n+1\equiv 2$ $(mod$ $p)$

 

Hai điều này mâu thuẫn nên chỉ có $n=1$ thỏa mãn điều kiện




#642353 $N=\frac{(a-b)(b-c)(c-a)}{2}+2$

Đã gửi bởi Minhnguyenthe333 on 26-06-2016 - 22:09 trong Số học

Bài toán: (JBMO 2016) Tìm tất cả bộ ba số nguyên $(a,b,c)$ để số: $N=\frac{(a-b)(b-c)(c-a)}{2}+2$

là một lũy thừa của $2016$.

Đặt $N=2016^k$ $(k$ là số tự nhiên)

Giả sử $a\geqslant b\geqslant c=>\left\{\begin{matrix}a-b\geqslant 0\\b-c\geqslant 0 \\ c-a\leqslant0\end{matrix}\right.$

Suy ra $(a-b)(b-c)(c-a)\leqslant 0<=>2016^k \leqslant 2=>k=0$

$<=>(a-b)(b-c)(c-a)=-2=1.1.(-2)=2.1.(-1)=>(a,b,c)=(1,0,-1)$ và các hoán vị

 

P/s: Lời giải mình thiếu TH $a\geqslant c\geqslant b$, khi đó $N\geqslant 2$




#642335 $f(n)=n$

Đã gửi bởi Minhnguyenthe333 on 26-06-2016 - 20:46 trong Số học

Cho $f:\mathbb{N^*}\rightarrow \mathbb{N^*}$ thỏa mãn điều kiện:

                      $f(n+1)>f(f(n))$ với mọi $n$ nguyên dương

Chứng minh rằng $f(n)=n$ với mọi $n$ nguyên dương




#642261 $n\mid 2^n-8$

Đã gửi bởi Minhnguyenthe333 on 26-06-2016 - 14:56 trong Số học

Chứng minh rằng tồn tại vô hạn số nguyên dương $n$ để: $n\mid 2^n-8$.

Spoiler

Ý tưởng là dùng định lý nhỏ Fermat
Ta cần chọn $n=kp$ với $p$ nguyên tố và $p>3$ sao cho:
$2^{kp}\equiv 2^3$ $(mod$ $p)$ và $2^{kp}\equiv 2^3$ $(mod$ $k)$
Chọn $k=3$ suy ra $8^p-8\equiv 0$ $(mod$ $p)$
Mặt khác $8^p-8\equiv 2^p-2\equiv 0$ $(mod$ $3)$ (do $p$ lẻ)
Lại có $(3,p)=1$ nên $3p\mid 8^p-8=2^{3p}-8$
mà có vô hạn số nguyên tố nên cũng có vô hạn số nguyên dương $n=3p$
Suy ra đpcm



#642169 Chứng minh rằng nếu a,b là 2 số nguyên dương thỏa mãn 4ab-1 là ước của (a+b-1...

Đã gửi bởi Minhnguyenthe333 on 25-06-2016 - 20:47 trong Số học

Chứng minh rằng nếu a,b là 2 số nguyên dương thỏa mãn 4ab-1 là ước của (a+b-1)(a+b+1) thì a=b

Ta có: $4ab-1 \mid (a+b-1)(a+b+1)<=>4ab-1\mid (a-b)^2+(4ab-1)$

$<=>4ab-1 \mid (a-b)^2$.Đến đây ta đặt $k_0=\frac{(a-b)^2}{4ab-1}$

 

$<=>2k_0+1=\frac{2(a^2+b^2)-1}{4ab-1}$.Ta tiếp túc đặt $k=2k_0+1=\frac{2(a^2+b^2)-1}{4ab-1}$

 

Với $k=1$ ta có $k_0=0$ kéo theo $a=b$

 

Với $k>1:$

$PT<=>2a^2-4kba+2b^2-1=0$ $(*)$. Giả sử $a\geqslant b$ và $a+b$ nhỏ nhất

 

Theo định lí Viet $PT(*)$ còn nghiệm $t$ sao cho: $\left\{\begin{matrix}t+a=2kb\\ ta=b^2-\frac{1}{2}\end{matrix}\right.$

 

Suy ra $t$ là số nguyên dương và do tính nhỏ nhất của $a+b$ nên $t\geqslant a\geqslant b$

$=>2kb=t+a\leqslant 2t<=>2kab\leqslant 2ta<=>2ta\geqslant 2kb^2$

$<=>2b^2-1\geqslant 2kb^2<=>2b^2(1-k)\geqslant 1$ (vô lí)

 

Vậy ta có đpcm




#642165 Tìm tất cả các số nguyên dương lẻ $n$ sao cho $n\mid 15^n...

Đã gửi bởi Minhnguyenthe333 on 25-06-2016 - 20:09 trong Số học

Tìm tất cả các số nguyên dương lẻ $n$ sao cho $n\mid 15^n+1$




#641939 Cho $x>0$. Chứng minh rằng $\sqrt{\dfrac...

Đã gửi bởi Minhnguyenthe333 on 23-06-2016 - 21:22 trong Bất đẳng thức và cực trị

Cho $x>0$. Chứng minh rằng $\sqrt{\dfrac{x}{2}+\dfrac{1}{2x}}\ge e^{\left(\frac{x-1}{x+1} \right)^2}$

BĐT$<=>\ln (\sqrt{\frac{x}{2}+\frac{1}{2x}})\geqslant \frac{(x-1)^2}{(x+1)^2}$ $(*)$

 

Đặt $f(x)=\ln (\sqrt{\frac{x}{2}+\frac{1}{2x}})- \frac{(x-1)^2}{(x+1)^2}$ $(x>0)$

 

Ta có: $f'(x)=\frac{x^2-1}{2x(x^2+1)}-\frac{4(x-1)}{(x+1)^3}=(x-1)[\frac{x+1}{2x(x^2+1)}-\frac{4}{(x+1)^3}]$

$f'(x)=0<=>x=1$ mà $f(x)$ đồng biến trên $x\in (0;\infty)$

 

Suy ra $f(x)\geqslant f(1)=0$ hay $\ln (\sqrt{\frac{x}{2}+\frac{1}{2x}})- \frac{(x-1)^2}{(x+1)^2}\geqslant 0$

 

Do đó $(*)$ luôn đúng nên ta có đpcm




#641230 Cho số nguyên n > 1 và số nguyên tố p sao cho p - 2

Đã gửi bởi Minhnguyenthe333 on 19-06-2016 - 10:51 trong Số học

Cho số nguyên n > 1 và số nguyên tố p sao cho p - 2 chia hết cho n và $n^{3}+n+2$ chia hết cho p . Chứng minh rằng 4p - 7 là một số chính phương

Ta có: $p\mid n^3+n+2<=>p\mid (n+1)(n^2-n+2)$

$TH1: n+1\mid p$

Do $n>1$ nên $n+1=p$

$=>n\mid p-2<=>\frac{p-2}{p-1}=1-\frac{1}{p-1}$ là số nguyên dương

$=>p=2$ kéo theo $4p-7=1$ là số chính phương

 

$TH2: p\mid n^2-n+2$

Suy ra $(\frac{p-2}{n})(\frac{n^2-n+2}{p})$ là số nguyên

 

$<=>(\frac{n^2}{p}-\frac{n}{p}+\frac{2}{p})(\frac{p}{n}-\frac{2}{n})$ $\in \mathbb{Z^+}$

 

$<=>n-1+\frac{2}{n}-\frac{2n}{p}+\frac{2}{p}-\frac{4}{pn}$ $\in \mathbb{Z^+}$

 

hay $\frac{2}{n}-\frac{2n}{p}+\frac{2}{p}-\frac{4}{pn}$ $\in \mathbb{Z^+}$

 

$<=>pn\mid 2(p+n-2-n^2)<=>pn \mid p+n-2-n^2$ (do $n$ là số lẻ)

$=>p+n-2-n^2\geqslant pn<=>n-n^2-2\geqslant p(n-1)$

Mặt khác do $n\mid p-2$ nên $p-2\geqslant n<=>p\geqslant n+2$

$=>n-n^2-2\geqslant p(n-1)\geqslant (n+2)(n-1)=n^2+n-2$

$<=>n^2\leqslant 0<=>n=0$ (vô lí) 

 

Vậy ta có đpcm




#641096 $x^{3}+y^{3}+z^{3}\leq 1+\frac...

Đã gửi bởi Minhnguyenthe333 on 18-06-2016 - 17:21 trong Bất đẳng thức và cực trị

Cho $x,y,z$ là các số thực không âm thoả mãn điều kiện $x+y+z=2.$ Chứng minh rằng: $$x^{3}+y^{3}+z^{3}\leq 1+\frac{1}{2}(x^{4}+y^{4}+z^{4}).$$

Đặt $a=xy+yz+zx$ và $b=xyz$
BĐT$<=>2(x^{3}+y^{3}+z^{3})\leq 2+x^{4}+y^{4}+z^{4}$

$<=>2[(x+y+z)^3-3(x+y+z)a+3b]\leqslant 2+[(x+y+z)^2-2a]^2-2[a^2-2b(x+y+z)]$

$<=>2(8-6a+3b)\leqslant 2+(4-2a)^2-2(a^2-4b)$

$<=>4a\leqslant 2+2b+2a^2$

$<=>2a\leqslant a^2+b+1<=>(a-1)^2+b\geqslant 0$ (luôn đúng do $x,y,z\geqslant 0$)

Dấu "=" xảy ra khi $(x,y,z)=(0,1,1)$ và các hoán vị



#641090 MIN: $H=a^3+b^3+c^3+\sqrt{5}abc$

Đã gửi bởi Minhnguyenthe333 on 18-06-2016 - 17:07 trong Bất đẳng thức và cực trị

Cho a,b,c dương thỏa mãn: $a+b+c=3$.
Tìm GTNN của $H=a^3+b^3+c^3+\sqrt{5}abc$

Biến đổi $(a+b+c,ab+bc+ca,abc)\rightarrow (p,q,r)$
Áp dụng bđt Schur: $r\geqslant \frac{p(4q-p^2)}{9}=\frac{4q-9}{3}$
và bđt $3q\leqslant p^2=9$
$H=p^3-3pq+3r+r\sqrt{5}=27-9q+(3+\sqrt{5})r\geqslant 27-9q+(3+\sqrt{5})\frac{4q-9}{3}$
$=18-3\sqrt{5}+\frac{-15+4\sqrt{5}}{3}q\geqslant 18-3\sqrt{5}-15+4\sqrt{5}=3+\sqrt{5}$
Dấu "=" xảy ra khi $a=b=c=1$



#640925 CMR: $xy+yz+zx\geq x+y+z$

Đã gửi bởi Minhnguyenthe333 on 17-06-2016 - 20:46 trong Bất đẳng thức và cực trị

Cho x,y,z dương thỏa mãn: $x+y+z+1=4xyz$.

Chứng minh rằng: $xy+yz+zx\geq x+y+z$




#640769 $k=\frac{x^2+y^2-1}{xy}$

Đã gửi bởi Minhnguyenthe333 on 16-06-2016 - 22:35 trong Số học

Ta có $x^2+y^2-1=kxy <=> x^2-kxy +y^2-1 =0 (*)$
Gọi $S$ là tập nghiệm của $(*)$
Ta có $1;1$ thuộc  $S$
Do đó $ S $ không rỗng 
Do đó, giả sử tồn tại nghiệm $x_0;y_0$ sao cho $x_0+y_0$ nhỏ nhất và $x_0 \geq y_0$
Ta đặt $f(x)=x^2  - kxy_0 +y_0^2 -1 $
Khi đó gọi nghiệm còn lại của pt là $x_1$
Bây giờ ta chứng minh $x_1$ nguyên dương
Thật vậy, ta có $x_0.x_1=y^2-1 => x_1$ dương
$x_0+x_1 = ky_0 => x_1$ nguyên
Do đó $x_1 $ nguyên dương 
Dễ thấy $x_1 \geq x_0 $ vì nếu $x_1 \leq x_0$ thì $x_1 +y_0 \geq x_0+y_0$ vô lí do $x_0+y_0$ nhỏ nhất
Do đó, ta có $x_1 \geq x_0 \geq y_0 $
Do đó $y_0$ nằm ngoài 2 nghiệm
Do đó $f(y_0) \geq 0 <=> (2-k)y_0^2 \geq 1 $
Do đó $k=1 $
Khi đó $x^2+y^2-1 =xy $ Tới đây dễ rồi

Bạn thiếu trường hợp $k=2$ thì khi đó $(x,y,k)=(t,t-1,2)$ với $t$ nguyên dương



#640676 $1+\sqrt{x+y+3}=\sqrt{x}+\sqrt{y}$

Đã gửi bởi Minhnguyenthe333 on 16-06-2016 - 12:29 trong Số học

Giải phương trình nghiệm nguyên:

Từ giả thiết suy ra $x=ta^2$ và $y=tb^2$,giả sử $a\geqslant b$

BÌnh phương 2 vế ta có:$PT<=>2=\sqrt{xy}-\sqrt{x+y+3}<=>2=tab-\sqrt{ta^2+tb^2+3}$

$<=>t^2a^2b^2-4tab+1=t(a^2+b^2)<=>ta^2b^2-4ab+\frac{1}{t}=a^2+b^2$

Dẫn đến $t=1:$ nên ta có:

$<=>ab-4=\frac{a^2+b^2-1}{ab}$ $(*)$

Đặt $k=\frac{a^2+b^2-1}{ab}$ $(k\in \mathbb{N^*})$

Dễ thấy $k=2$ thì $x=9$ và $y=4$

Xét $k\neq 2$:

$PT<=>a^2-kba+b^2-1=0$>Suy ra $a$ là nghiệm của phương trình $X^2-X.kb+b^2-1=0$ $(1)$

Cố định tập nghiệm.Chọn cặp $(a,b)$ sao $a+b$ nhỏ nhất 

Ngoài nghiệm $a$ thì $PT(1)$ còn nghiệm $x_0$ nên theo $Viete$

$\left\{\begin{matrix}x_0+a=kb\\ ax_0=b^2-1 \end{matrix}\right.$

Suy ra $x_0$ nguyên dương nên theo tính nhỏ nhất của $a+b=>x_0\geqslant a$

$<=>kb=x_0+a\leqslant 2x_0<=>2ax_0\geqslant kab\geqslant kb^2$

$=>2(b^2-1)=2ax_0\geqslant  kb^2<=>b^2(2-k)\geqslant 2$

$<=>k=1<=>ab=5=>PT$ vô nghiệm

Vậy $(x,y)=(9,4)$ và hoán vị




#640647 $k=\frac{x^2+y^2-1}{xy}$

Đã gửi bởi Minhnguyenthe333 on 16-06-2016 - 10:34 trong Số học

Tìm $x,y$ nguyên dương để $k=\frac{x^2+y^2-1}{xy}$ là số nguyên dương 




#640469 Chờ m,n,p là độ dài 3 cạnh tam giác có chu vi 2 CMR: $\frac{52...

Đã gửi bởi Minhnguyenthe333 on 15-06-2016 - 12:01 trong Bất đẳng thức và cực trị

Chờ a,b,c là độ dài 3 cạnh tam giác có chu vi 2

CMR: $\frac{52}{27}\leq a^{2}+b^{2}+c^{2}+2abc$

Ta có: $2-2a=a+b+c-2a=b+c-a>0$

Tương tự cho $(2-2b),(2-2c)$. Áp dụng bđt Cauchy:

$(2-2a)(2-2b)(2-2c)\leqslant \frac{[6-2(a+b+c)]^3}{27}$

$<=>8-8(a+b+c)+8(ab+bc+ca)-8abc\leqslant \frac{8}{27}$

$<=>-\frac{224}{27}\leqslant 8abc-8(ab+bc+ca)$

$<=>-\frac{224}{27}\leqslant 8abc-4[(a+b+c)^2-(a^2+b^2+c^2)]$

$<=>\frac{208}{27}\leqslant 4(a^2+b^2+c^2)+8abc$

$<=>a^2+b^2+c^2+2abc\geqslant \frac{52}{27}$ (đpcm)

Dấu "=" xảy ra khi $a=b=c=\frac{2}{3}$




#640345 $A=3(\frac{x^2}{y^2}+\frac{y^2}...

Đã gửi bởi Minhnguyenthe333 on 14-06-2016 - 20:59 trong Bất đẳng thức và cực trị

Cho x,y là các số thức khác 0. Tìm GTNN của biểu thức

$A=3(\frac{x^2}{y^2}+\frac{y^2}{x^2})-8(\frac{x}{y}+\frac{y}{x})$

Đặt $t=\frac{x}{y}+\frac{y}{x}$

$=>A=3(t^2-2)-8t=3t^2-8t-6=(\sqrt{3}t-\frac{8}{2\sqrt{3}})^2-\frac{34}{3}\geqslant -\frac{34}{3}$




#640261 $k=\frac{a^{2}+b^{2}}{ab-1}...

Đã gửi bởi Minhnguyenthe333 on 14-06-2016 - 14:31 trong Số học

Cho $a,b$ là các số nguyên thoả mãn $k=\frac{a^{2}+b^{2}}{ab-1}$ là số nguyên. Chứng minh rằng: $k=5$.

Đề bài hình như là phải thêm điều kiện $a,b\neq 0$
Đặt $x=a+b$ và $y=a-b$
Từ $k=\frac{a^{2}+b^{2}}{ab-1}=>\frac{x^2+y^2}{x^2-y^2-4}=\frac{k}{2}$ $(*)$
$TH1:a$ chẵn và $b$ lẻ$=>k$ là số lẻ
Đặt $d=(x^2+y^2,x^2-y^2-4)$
$=>d=(2x^2-4,2y^2+4)<=>d\mid 2(x^2+y^2)=4(a^2+b^2)$ $(1)$
Mặt khác từ $(*)$ ta cũng có: $\left\{\begin{matrix}x^2+y^2=kd \\ x^2-y^2-4=2d\end{matrix}\right.$
$<=>2x^2-4=d(k+2)<=>2\mid d$ $(2)$
mà $a^2+b^2$ là số lẻ nên từ $(1),(2)$ suy ra $d=2$ hoặc $d=4$
Thay $d=2$ hoặc $d=4$ vào hệ trên và áp dụng giả thiết$=>(a,b)=(\pm 2,\pm 1)$ và $k=5$

$TH2: a,b$ lẻ$=>a=2m+1$ và $b=2n+1<=>k=\frac{2(m^2+n^2+m+n)+1}{2mn+m+n}$ là số lẻ
Tương tự như trên ta có: $d\mid 4(a^2+b^2)=8(2m^2+2n^2+2m+2n+1)$
Và $2x^2-4=(k+2)d<=>2\mid d$ nên $d\in \{2;4;8\}$
Thay vào hệ trên suy ra $(a,b)=(\pm 3,\pm 1)$ và $k=5$
Vậy ta có đpcm



#640246 $A=\dfrac{1}{x+y+z+1}-\dfrac{1}...

Đã gửi bởi Minhnguyenthe333 on 14-06-2016 - 11:15 trong Bất đẳng thức và cực trị

Cho $x,y,z>0$ tìm min của:

$A=\dfrac{1}{x+y+z+1}-\dfrac{1}{(x+1)(y+1)(z+1)}$

Bài này $A$ không tồn tại $GTNN$ vì khi cho $x,y,z\rightarrow \infty$ thì $A$ luôn đạt một giá trị $k_{n+1}<k_n$.Do đó nếu giả thiết cho $x,y,z\geqslant 0$ thì $A_{min}=0$.Mặt khác $A$ có $GTLN$ là $A_{max}=\frac{1}{8}$
Thật vậy:
$A\leqslant \frac{1}{x+y+z+1}-\frac{27}{(x+y+z+3)^3}=\frac{1}{t+1}-\frac{27}{(t+3)^3}$ $(t=x+y+z)$
Đặt $f(t)=\frac{1}{t+1}-\frac{27}{(t+3)^3}$
Khảo sát hàm số trên, có đạo hàm $f'(t)=\frac{t(t-3)(t^2+15t+18)}{(t+3)^4(t+1)^2}$
$=>f'(t)=0<=>..........<=>f(t)\leqslant f(3)=\frac{1}{8}$
Dấu "=" xảy ra khi $x=y=z=1$